¿Por qué la descomposición de un Hamiltoniano qubit-qutrit en términos de matrices de Pauli y Gell-Mann no es única?


7

Si tengo la puerta X que actúa en un qubit y la puerta λ6 que actúa en un qutrit, donde es una matriz de Gell-Mann , el sistema está sujeto al Hamiltoniano:λ6

λ6X=(000000000000000001000010000100001000)

En caso de que alguien dude de esta matriz, se puede generar con el siguiente script (MATLAB / octava):

lambda6=[0 0 0; 0 0 1; 0 1 0];
X=      [0 1; 1 0 ];
kron(lambda6,X)

Sin embargo, considere la alternativa hamiltoniana:

12Zλ1+12λ113Xλ8+13X .

¡Esto es exactamente el mismo hamiltoniano!

El siguiente script lo demuestra:

lambda1=[0 1 0;1 0 0;0 0 0];
lambda8=[1 0 0;0 1 0;0 0 -2]/sqrt(3);
Z=      [1 0; 0 -1 ];
round(-0.5*kron(Z,lambda1)+0.5*kron(eye(2),lambda1)-(1/sqrt(3))*kron(X,lambda8)+(1/3)*kron(X,eye(3)))

La "ronda" en la última línea de código se puede eliminar, pero el formato será más feo porque algunos de los 0 terminan siendo alrededor de .1016

1) Pensé que la descomposición de Pauli para dos qubits es única, ¿por qué la descomposición de Pauli-GellMann de un qubit-qutrit no es única?

2) ¿Cómo obtendría la descomposición de la matriz 6x6 anterior?λ6X

Respuestas:


5

Obtiene dos descomposiciones para su matriz (llamémosla ) porque está utilizando dos bases operativas diferentes.A

En el primer caso, está considerando que la matriz actúa en un espacio de dimensión , es decir, utilizando la base operativa { λ i σ j } i j{ λ iσ j } i j .3×2{λiσj}ij{λiσj}ij

En otras palabras, estás calculando los coeficientes , encontrando que c 61 es el único término que no desaparece. Esta descomposición será única, porque tr [ ( λ i σ j ) ( λ k σ l ) ] = N i j δ i k δ j l .cij=tr((λiσj)A)c61tr[(λiσj)(λkσl)]=Nijδikδjl

Por otro lado, la segunda descomposición se obtiene pensando en como una matriz en un espacio de dimensiones 2 × 3 , es decir, descomponiéndola utilizando la base operativa { σ i λ j } i j{ σ iλ j } i j . Esto le da nuevos coeficientes d i jtr ( ( σ iλ j ) A ) , que no tienen que ser (y de hecho no son) iguales a los cA2×3{σiλj}ij{σiλj}ijdijtr((σiλj)A) .cij

No hay paradoja porque y { λ iσ j } i j son dos bases operativas completamente diferentes para un espacio de dimensión 6 .{σiλj}ij{λiσj}ij6


Creo que esta es la respuesta correcta: simplemente que las dos descomposiciones están en bases diferentes, que es a lo que aludí en mi comentario a la otra respuesta: en un caso, actúa primero en el qubit y luego en el qutrit, y en el otro caso Es al revés (diferentes bases). Podría haberme confundido porque hasta hace poco trabajaba casi exclusivamente con hamiltonianos que contenían matrices Z (modelos Ising), y todo se conmuta allí, por lo que este problema nunca surgió.
user1271772

4

Esto se ve esencialmente similar a la propiedad de no conmutatividad del producto Kronecker: :Xλ6λ6X

Xλ6=(0110)(000001010)=(000000000001000010000000001000010000)

Como era de esperar, no se puede descomponer enXλ6.12Zλ1+12I2λ113Xλ8+13XI3=λ6XXλ6

Sin embargo, como ambas matrices son cuadradas, son 'permutación similar', de modo que para alguna matriz de permutación PXλ6=PT(λ6X)P P

En otras palabras, para responder a la parte 1, para una permutación / ordenamiento dada, la descomposición es única, pero cuando se cambia el ordenamiento, la matriz / hamiltoniana sufre una rotación , que también cambia la descomposición.(PT=P1)

Queda claro qué se puede usar para descomponer una matriz de esta forma dividiéndola en submatrices: escribiendo donde cada submatriz A , B , C y D es un 3 × 3 matriz, queda claro que A = D = 0 y B = C = λ 6 , que verifica X λ 6 = ( 0 λ 6 λ

Xλ6=(ABCD),
A,B,CD3×3A=D=0B=C=λ6
Xλ6=(0λ6λ60)=Xλ6

Realizar la rotación / permutar y aplicar la misma idea da que da que A = 0 ,

M=(000000000000000001000010000100001000)=(ABCD),
A=0,B=C=(000000001),D=(010100000)=λ1

B=C=13I313λ8

M=(013I313λ813I313λ8λ1)=12(IZ)λ1+X(13I313λ8).

Changing the order of the decomposition:

M=(ABCDEFGHJ),
which gives A=B=C=D=E=G=J=0 and F=H=X, in turn giving
M=(00000X0X0)=λ6X

Supongo que esto responde la pregunta: λ6 6X acts on the qutrir first then the qubit, whereas the other expression acts on the qubit first then the qutrir, but I still don't get why there's two decompositions because working with only qubits I've never seen something like this. I hate to edit the question after you did all this work, but the way it's written (which I apologize you already spent time answering) is wrong, because as you said, Xλ6 is not the matrix I have there :'(
user1271772

@user1271772 I'm not sure I understand: does this answer your question, after the typo was fixed?
glS

1
C2C3C6C3C2 but there is data in this isomorphism. Not canonical. Think with categories.
AHusain
Al usar nuestro sitio, usted reconoce que ha leído y comprende nuestra Política de Cookies y Política de Privacidad.
Licensed under cc by-sa 3.0 with attribution required.